Prove that $frac{a}{csqrt{a^2+1}}+frac{b}{asqrt{b^2+1}}+frac{c}{bsqrt{c^2+1}}ge frac{3}{2}$












1












$begingroup$


Let $a,b,cin Bbb R^+$ such that $a+b+c=abc$. Prove that $$frac{a}{csqrt{a^2+1}}+frac{b}{asqrt{b^2+1}}+frac{c}{bsqrt{c^2+1}}ge frac{3}{2}$$





Idea 1.From $a+b+c=abcLeftrightarrow frac{1}{ab}+frac{1}{bc}+frac{1}{ca}=1$. Let $left(frac{1}{a};frac{1}{b};frac{1}{c}right)rightarrow left(x;y;zright)$



So i need to prove $frac{z}{sqrt{1+x^2}}+frac{y}{sqrt{z^2+1}}+frac{x}{sqrt{y^2+1}}ge frac{3}{2}$



By AM-GM $frac{x}{sqrt{y^2+1}}=frac{x}{sqrt{left(x+yright)left(y+zright)}}ge frac{2x}{x+2y+z}$



$$LHSge 2sum _{cyc}frac{x}{x+2y+z}=2sum _{cyc}frac{x^2}{x^2+2xy+xz}ge 2frac{left(x+y+zright)^2}{sum _{cyc}x^2+sum _{cyc}3xy}$$



Or $4left(x+y+zright)^2ge 3left(x^2+y^2+z^2+3xy+3yz+3xzright)$



Or $x^2+y^2+z^2ge xy+yz+xz$ (true)



Idea 2. By Holder $$left(sum _{cyc}frac{a}{csqrt{a^2+1}}right)left(sum _{cyc}frac{a}{csqrt{a^2+1}}right)sum _{cyc}left(c^2left(a^2+1right)aright)ge left(sum _{cyc}aright)^3$$



I will prove the inequality $frac{left(a+b+cright)^3}{c^2aleft(a^2+1right)+a^2bleft(b^2+1right)+b^2cleft(c^2+1right)}ge frac{3}{2}$



Or $frac{abcleft(a+b+cright)^3}{left(a+b+cright)left(a^2b^3+b^2c^3+c^2a^3right)+abcleft(a^2b+b^2c+c^2aright)}ge frac{3}{2}$



I tried $SOS$ but failed help me improve "idea 2" use Holder










share|cite|improve this question











$endgroup$

















    1












    $begingroup$


    Let $a,b,cin Bbb R^+$ such that $a+b+c=abc$. Prove that $$frac{a}{csqrt{a^2+1}}+frac{b}{asqrt{b^2+1}}+frac{c}{bsqrt{c^2+1}}ge frac{3}{2}$$





    Idea 1.From $a+b+c=abcLeftrightarrow frac{1}{ab}+frac{1}{bc}+frac{1}{ca}=1$. Let $left(frac{1}{a};frac{1}{b};frac{1}{c}right)rightarrow left(x;y;zright)$



    So i need to prove $frac{z}{sqrt{1+x^2}}+frac{y}{sqrt{z^2+1}}+frac{x}{sqrt{y^2+1}}ge frac{3}{2}$



    By AM-GM $frac{x}{sqrt{y^2+1}}=frac{x}{sqrt{left(x+yright)left(y+zright)}}ge frac{2x}{x+2y+z}$



    $$LHSge 2sum _{cyc}frac{x}{x+2y+z}=2sum _{cyc}frac{x^2}{x^2+2xy+xz}ge 2frac{left(x+y+zright)^2}{sum _{cyc}x^2+sum _{cyc}3xy}$$



    Or $4left(x+y+zright)^2ge 3left(x^2+y^2+z^2+3xy+3yz+3xzright)$



    Or $x^2+y^2+z^2ge xy+yz+xz$ (true)



    Idea 2. By Holder $$left(sum _{cyc}frac{a}{csqrt{a^2+1}}right)left(sum _{cyc}frac{a}{csqrt{a^2+1}}right)sum _{cyc}left(c^2left(a^2+1right)aright)ge left(sum _{cyc}aright)^3$$



    I will prove the inequality $frac{left(a+b+cright)^3}{c^2aleft(a^2+1right)+a^2bleft(b^2+1right)+b^2cleft(c^2+1right)}ge frac{3}{2}$



    Or $frac{abcleft(a+b+cright)^3}{left(a+b+cright)left(a^2b^3+b^2c^3+c^2a^3right)+abcleft(a^2b+b^2c+c^2aright)}ge frac{3}{2}$



    I tried $SOS$ but failed help me improve "idea 2" use Holder










    share|cite|improve this question











    $endgroup$















      1












      1








      1


      1



      $begingroup$


      Let $a,b,cin Bbb R^+$ such that $a+b+c=abc$. Prove that $$frac{a}{csqrt{a^2+1}}+frac{b}{asqrt{b^2+1}}+frac{c}{bsqrt{c^2+1}}ge frac{3}{2}$$





      Idea 1.From $a+b+c=abcLeftrightarrow frac{1}{ab}+frac{1}{bc}+frac{1}{ca}=1$. Let $left(frac{1}{a};frac{1}{b};frac{1}{c}right)rightarrow left(x;y;zright)$



      So i need to prove $frac{z}{sqrt{1+x^2}}+frac{y}{sqrt{z^2+1}}+frac{x}{sqrt{y^2+1}}ge frac{3}{2}$



      By AM-GM $frac{x}{sqrt{y^2+1}}=frac{x}{sqrt{left(x+yright)left(y+zright)}}ge frac{2x}{x+2y+z}$



      $$LHSge 2sum _{cyc}frac{x}{x+2y+z}=2sum _{cyc}frac{x^2}{x^2+2xy+xz}ge 2frac{left(x+y+zright)^2}{sum _{cyc}x^2+sum _{cyc}3xy}$$



      Or $4left(x+y+zright)^2ge 3left(x^2+y^2+z^2+3xy+3yz+3xzright)$



      Or $x^2+y^2+z^2ge xy+yz+xz$ (true)



      Idea 2. By Holder $$left(sum _{cyc}frac{a}{csqrt{a^2+1}}right)left(sum _{cyc}frac{a}{csqrt{a^2+1}}right)sum _{cyc}left(c^2left(a^2+1right)aright)ge left(sum _{cyc}aright)^3$$



      I will prove the inequality $frac{left(a+b+cright)^3}{c^2aleft(a^2+1right)+a^2bleft(b^2+1right)+b^2cleft(c^2+1right)}ge frac{3}{2}$



      Or $frac{abcleft(a+b+cright)^3}{left(a+b+cright)left(a^2b^3+b^2c^3+c^2a^3right)+abcleft(a^2b+b^2c+c^2aright)}ge frac{3}{2}$



      I tried $SOS$ but failed help me improve "idea 2" use Holder










      share|cite|improve this question











      $endgroup$




      Let $a,b,cin Bbb R^+$ such that $a+b+c=abc$. Prove that $$frac{a}{csqrt{a^2+1}}+frac{b}{asqrt{b^2+1}}+frac{c}{bsqrt{c^2+1}}ge frac{3}{2}$$





      Idea 1.From $a+b+c=abcLeftrightarrow frac{1}{ab}+frac{1}{bc}+frac{1}{ca}=1$. Let $left(frac{1}{a};frac{1}{b};frac{1}{c}right)rightarrow left(x;y;zright)$



      So i need to prove $frac{z}{sqrt{1+x^2}}+frac{y}{sqrt{z^2+1}}+frac{x}{sqrt{y^2+1}}ge frac{3}{2}$



      By AM-GM $frac{x}{sqrt{y^2+1}}=frac{x}{sqrt{left(x+yright)left(y+zright)}}ge frac{2x}{x+2y+z}$



      $$LHSge 2sum _{cyc}frac{x}{x+2y+z}=2sum _{cyc}frac{x^2}{x^2+2xy+xz}ge 2frac{left(x+y+zright)^2}{sum _{cyc}x^2+sum _{cyc}3xy}$$



      Or $4left(x+y+zright)^2ge 3left(x^2+y^2+z^2+3xy+3yz+3xzright)$



      Or $x^2+y^2+z^2ge xy+yz+xz$ (true)



      Idea 2. By Holder $$left(sum _{cyc}frac{a}{csqrt{a^2+1}}right)left(sum _{cyc}frac{a}{csqrt{a^2+1}}right)sum _{cyc}left(c^2left(a^2+1right)aright)ge left(sum _{cyc}aright)^3$$



      I will prove the inequality $frac{left(a+b+cright)^3}{c^2aleft(a^2+1right)+a^2bleft(b^2+1right)+b^2cleft(c^2+1right)}ge frac{3}{2}$



      Or $frac{abcleft(a+b+cright)^3}{left(a+b+cright)left(a^2b^3+b^2c^3+c^2a^3right)+abcleft(a^2b+b^2c+c^2aright)}ge frac{3}{2}$



      I tried $SOS$ but failed help me improve "idea 2" use Holder







      inequality radicals geometric-inequalities holder-inequality rearrangement-inequality






      share|cite|improve this question















      share|cite|improve this question













      share|cite|improve this question




      share|cite|improve this question








      edited Jan 8 at 19:09









      Michael Rozenberg

      100k1591193




      100k1591193










      asked Jan 8 at 15:07









      Nguyễn Duy LinhNguyễn Duy Linh

      1818




      1818






















          1 Answer
          1






          active

          oldest

          votes


















          2












          $begingroup$

          We need to prove that
          $$sum_{cyc}frac{a}{csqrt{a^2+frac{abc}{a+b+c}}}geqfrac{3}{2}$$ or
          $$sum_{cyc}sqrt{frac{a(a+b+c)}{c^2(a+b)(a+c)}}geqfrac{3}{2}$$ or
          $$sum_{cyc}sqrt{frac{a^2b}{c(a+b)(a+c)}}geqfrac{3}{2}$$ or
          $$sum_{cyc }sqrt{a^3b^2(b+c)}geqfrac{3}{2}sqrt{abc(a+b)(a+c)(b+c)}$$ or
          $$sum_{cyc }sqrt{(a^3b^3+a^3b^2c)}geqfrac{3}{2}sqrt{abc(a+b)(a+c)(b+c)}.$$
          Now, let $ab=z$, $ac=y$ and $bc=x$.



          Thus, we need to prove that
          $$sum_{cyc}sqrt{z^3+z^2x}geqfrac{3}{2}sqrt{(x+y)(x+z)(y+z)}.$$
          By Holder
          $$left(sum_{cyc}sqrt{z^3+z^2x}right)^2sum_{cyc}frac{z}{z+x}geq(x+y+z)^3.$$
          Id est, it's enough to prove that
          $$4(x+y+z)^3geq9prod_{cyc}(x+y)sum_{cyc}frac{z}{z+x}$$ or
          $$4(x+y+z)^3geq9sum_{cyc}x(x+z)(y+z)$$ or
          $$4(x+y+z)^3geq9sum_{cyc}(2x^2y+x^2z+xyz).$$
          Now, by Rearrangement easy to show that $$x^2y+y^2z+z^2x+xyzleqfrac{4}{27}(x+y+z)^3.$$
          Thus, it's enough to prove that
          $$4(x+y+z)^3geqfrac{4}{3}(x+y+z)^3+9sum_{cyc}left(x^2y+x^2z+frac{2}{3}xyzright)$$ or
          $$frac{8}{3}(x+y+z)^3geq9(x+y)(x+z)(y+z)$$ or
          $$left(frac{x+y+x+z+y+z}{3}right)^3geq(x+y)(x+z)(y+z),$$ which is true by AM-GM.



          Done!






          share|cite|improve this answer











          $endgroup$













            Your Answer





            StackExchange.ifUsing("editor", function () {
            return StackExchange.using("mathjaxEditing", function () {
            StackExchange.MarkdownEditor.creationCallbacks.add(function (editor, postfix) {
            StackExchange.mathjaxEditing.prepareWmdForMathJax(editor, postfix, [["$", "$"], ["\\(","\\)"]]);
            });
            });
            }, "mathjax-editing");

            StackExchange.ready(function() {
            var channelOptions = {
            tags: "".split(" "),
            id: "69"
            };
            initTagRenderer("".split(" "), "".split(" "), channelOptions);

            StackExchange.using("externalEditor", function() {
            // Have to fire editor after snippets, if snippets enabled
            if (StackExchange.settings.snippets.snippetsEnabled) {
            StackExchange.using("snippets", function() {
            createEditor();
            });
            }
            else {
            createEditor();
            }
            });

            function createEditor() {
            StackExchange.prepareEditor({
            heartbeatType: 'answer',
            autoActivateHeartbeat: false,
            convertImagesToLinks: true,
            noModals: true,
            showLowRepImageUploadWarning: true,
            reputationToPostImages: 10,
            bindNavPrevention: true,
            postfix: "",
            imageUploader: {
            brandingHtml: "Powered by u003ca class="icon-imgur-white" href="https://imgur.com/"u003eu003c/au003e",
            contentPolicyHtml: "User contributions licensed under u003ca href="https://creativecommons.org/licenses/by-sa/3.0/"u003ecc by-sa 3.0 with attribution requiredu003c/au003e u003ca href="https://stackoverflow.com/legal/content-policy"u003e(content policy)u003c/au003e",
            allowUrls: true
            },
            noCode: true, onDemand: true,
            discardSelector: ".discard-answer"
            ,immediatelyShowMarkdownHelp:true
            });


            }
            });














            draft saved

            draft discarded


















            StackExchange.ready(
            function () {
            StackExchange.openid.initPostLogin('.new-post-login', 'https%3a%2f%2fmath.stackexchange.com%2fquestions%2f3066277%2fprove-that-fracac-sqrta21-fracba-sqrtb21-fraccb-sqrtc2%23new-answer', 'question_page');
            }
            );

            Post as a guest















            Required, but never shown

























            1 Answer
            1






            active

            oldest

            votes








            1 Answer
            1






            active

            oldest

            votes









            active

            oldest

            votes






            active

            oldest

            votes









            2












            $begingroup$

            We need to prove that
            $$sum_{cyc}frac{a}{csqrt{a^2+frac{abc}{a+b+c}}}geqfrac{3}{2}$$ or
            $$sum_{cyc}sqrt{frac{a(a+b+c)}{c^2(a+b)(a+c)}}geqfrac{3}{2}$$ or
            $$sum_{cyc}sqrt{frac{a^2b}{c(a+b)(a+c)}}geqfrac{3}{2}$$ or
            $$sum_{cyc }sqrt{a^3b^2(b+c)}geqfrac{3}{2}sqrt{abc(a+b)(a+c)(b+c)}$$ or
            $$sum_{cyc }sqrt{(a^3b^3+a^3b^2c)}geqfrac{3}{2}sqrt{abc(a+b)(a+c)(b+c)}.$$
            Now, let $ab=z$, $ac=y$ and $bc=x$.



            Thus, we need to prove that
            $$sum_{cyc}sqrt{z^3+z^2x}geqfrac{3}{2}sqrt{(x+y)(x+z)(y+z)}.$$
            By Holder
            $$left(sum_{cyc}sqrt{z^3+z^2x}right)^2sum_{cyc}frac{z}{z+x}geq(x+y+z)^3.$$
            Id est, it's enough to prove that
            $$4(x+y+z)^3geq9prod_{cyc}(x+y)sum_{cyc}frac{z}{z+x}$$ or
            $$4(x+y+z)^3geq9sum_{cyc}x(x+z)(y+z)$$ or
            $$4(x+y+z)^3geq9sum_{cyc}(2x^2y+x^2z+xyz).$$
            Now, by Rearrangement easy to show that $$x^2y+y^2z+z^2x+xyzleqfrac{4}{27}(x+y+z)^3.$$
            Thus, it's enough to prove that
            $$4(x+y+z)^3geqfrac{4}{3}(x+y+z)^3+9sum_{cyc}left(x^2y+x^2z+frac{2}{3}xyzright)$$ or
            $$frac{8}{3}(x+y+z)^3geq9(x+y)(x+z)(y+z)$$ or
            $$left(frac{x+y+x+z+y+z}{3}right)^3geq(x+y)(x+z)(y+z),$$ which is true by AM-GM.



            Done!






            share|cite|improve this answer











            $endgroup$


















              2












              $begingroup$

              We need to prove that
              $$sum_{cyc}frac{a}{csqrt{a^2+frac{abc}{a+b+c}}}geqfrac{3}{2}$$ or
              $$sum_{cyc}sqrt{frac{a(a+b+c)}{c^2(a+b)(a+c)}}geqfrac{3}{2}$$ or
              $$sum_{cyc}sqrt{frac{a^2b}{c(a+b)(a+c)}}geqfrac{3}{2}$$ or
              $$sum_{cyc }sqrt{a^3b^2(b+c)}geqfrac{3}{2}sqrt{abc(a+b)(a+c)(b+c)}$$ or
              $$sum_{cyc }sqrt{(a^3b^3+a^3b^2c)}geqfrac{3}{2}sqrt{abc(a+b)(a+c)(b+c)}.$$
              Now, let $ab=z$, $ac=y$ and $bc=x$.



              Thus, we need to prove that
              $$sum_{cyc}sqrt{z^3+z^2x}geqfrac{3}{2}sqrt{(x+y)(x+z)(y+z)}.$$
              By Holder
              $$left(sum_{cyc}sqrt{z^3+z^2x}right)^2sum_{cyc}frac{z}{z+x}geq(x+y+z)^3.$$
              Id est, it's enough to prove that
              $$4(x+y+z)^3geq9prod_{cyc}(x+y)sum_{cyc}frac{z}{z+x}$$ or
              $$4(x+y+z)^3geq9sum_{cyc}x(x+z)(y+z)$$ or
              $$4(x+y+z)^3geq9sum_{cyc}(2x^2y+x^2z+xyz).$$
              Now, by Rearrangement easy to show that $$x^2y+y^2z+z^2x+xyzleqfrac{4}{27}(x+y+z)^3.$$
              Thus, it's enough to prove that
              $$4(x+y+z)^3geqfrac{4}{3}(x+y+z)^3+9sum_{cyc}left(x^2y+x^2z+frac{2}{3}xyzright)$$ or
              $$frac{8}{3}(x+y+z)^3geq9(x+y)(x+z)(y+z)$$ or
              $$left(frac{x+y+x+z+y+z}{3}right)^3geq(x+y)(x+z)(y+z),$$ which is true by AM-GM.



              Done!






              share|cite|improve this answer











              $endgroup$
















                2












                2








                2





                $begingroup$

                We need to prove that
                $$sum_{cyc}frac{a}{csqrt{a^2+frac{abc}{a+b+c}}}geqfrac{3}{2}$$ or
                $$sum_{cyc}sqrt{frac{a(a+b+c)}{c^2(a+b)(a+c)}}geqfrac{3}{2}$$ or
                $$sum_{cyc}sqrt{frac{a^2b}{c(a+b)(a+c)}}geqfrac{3}{2}$$ or
                $$sum_{cyc }sqrt{a^3b^2(b+c)}geqfrac{3}{2}sqrt{abc(a+b)(a+c)(b+c)}$$ or
                $$sum_{cyc }sqrt{(a^3b^3+a^3b^2c)}geqfrac{3}{2}sqrt{abc(a+b)(a+c)(b+c)}.$$
                Now, let $ab=z$, $ac=y$ and $bc=x$.



                Thus, we need to prove that
                $$sum_{cyc}sqrt{z^3+z^2x}geqfrac{3}{2}sqrt{(x+y)(x+z)(y+z)}.$$
                By Holder
                $$left(sum_{cyc}sqrt{z^3+z^2x}right)^2sum_{cyc}frac{z}{z+x}geq(x+y+z)^3.$$
                Id est, it's enough to prove that
                $$4(x+y+z)^3geq9prod_{cyc}(x+y)sum_{cyc}frac{z}{z+x}$$ or
                $$4(x+y+z)^3geq9sum_{cyc}x(x+z)(y+z)$$ or
                $$4(x+y+z)^3geq9sum_{cyc}(2x^2y+x^2z+xyz).$$
                Now, by Rearrangement easy to show that $$x^2y+y^2z+z^2x+xyzleqfrac{4}{27}(x+y+z)^3.$$
                Thus, it's enough to prove that
                $$4(x+y+z)^3geqfrac{4}{3}(x+y+z)^3+9sum_{cyc}left(x^2y+x^2z+frac{2}{3}xyzright)$$ or
                $$frac{8}{3}(x+y+z)^3geq9(x+y)(x+z)(y+z)$$ or
                $$left(frac{x+y+x+z+y+z}{3}right)^3geq(x+y)(x+z)(y+z),$$ which is true by AM-GM.



                Done!






                share|cite|improve this answer











                $endgroup$



                We need to prove that
                $$sum_{cyc}frac{a}{csqrt{a^2+frac{abc}{a+b+c}}}geqfrac{3}{2}$$ or
                $$sum_{cyc}sqrt{frac{a(a+b+c)}{c^2(a+b)(a+c)}}geqfrac{3}{2}$$ or
                $$sum_{cyc}sqrt{frac{a^2b}{c(a+b)(a+c)}}geqfrac{3}{2}$$ or
                $$sum_{cyc }sqrt{a^3b^2(b+c)}geqfrac{3}{2}sqrt{abc(a+b)(a+c)(b+c)}$$ or
                $$sum_{cyc }sqrt{(a^3b^3+a^3b^2c)}geqfrac{3}{2}sqrt{abc(a+b)(a+c)(b+c)}.$$
                Now, let $ab=z$, $ac=y$ and $bc=x$.



                Thus, we need to prove that
                $$sum_{cyc}sqrt{z^3+z^2x}geqfrac{3}{2}sqrt{(x+y)(x+z)(y+z)}.$$
                By Holder
                $$left(sum_{cyc}sqrt{z^3+z^2x}right)^2sum_{cyc}frac{z}{z+x}geq(x+y+z)^3.$$
                Id est, it's enough to prove that
                $$4(x+y+z)^3geq9prod_{cyc}(x+y)sum_{cyc}frac{z}{z+x}$$ or
                $$4(x+y+z)^3geq9sum_{cyc}x(x+z)(y+z)$$ or
                $$4(x+y+z)^3geq9sum_{cyc}(2x^2y+x^2z+xyz).$$
                Now, by Rearrangement easy to show that $$x^2y+y^2z+z^2x+xyzleqfrac{4}{27}(x+y+z)^3.$$
                Thus, it's enough to prove that
                $$4(x+y+z)^3geqfrac{4}{3}(x+y+z)^3+9sum_{cyc}left(x^2y+x^2z+frac{2}{3}xyzright)$$ or
                $$frac{8}{3}(x+y+z)^3geq9(x+y)(x+z)(y+z)$$ or
                $$left(frac{x+y+x+z+y+z}{3}right)^3geq(x+y)(x+z)(y+z),$$ which is true by AM-GM.



                Done!







                share|cite|improve this answer














                share|cite|improve this answer



                share|cite|improve this answer








                edited Jan 8 at 19:06

























                answered Jan 8 at 19:01









                Michael RozenbergMichael Rozenberg

                100k1591193




                100k1591193






























                    draft saved

                    draft discarded




















































                    Thanks for contributing an answer to Mathematics Stack Exchange!


                    • Please be sure to answer the question. Provide details and share your research!

                    But avoid



                    • Asking for help, clarification, or responding to other answers.

                    • Making statements based on opinion; back them up with references or personal experience.


                    Use MathJax to format equations. MathJax reference.


                    To learn more, see our tips on writing great answers.




                    draft saved


                    draft discarded














                    StackExchange.ready(
                    function () {
                    StackExchange.openid.initPostLogin('.new-post-login', 'https%3a%2f%2fmath.stackexchange.com%2fquestions%2f3066277%2fprove-that-fracac-sqrta21-fracba-sqrtb21-fraccb-sqrtc2%23new-answer', 'question_page');
                    }
                    );

                    Post as a guest















                    Required, but never shown





















































                    Required, but never shown














                    Required, but never shown












                    Required, but never shown







                    Required, but never shown

































                    Required, but never shown














                    Required, but never shown












                    Required, but never shown







                    Required, but never shown







                    Popular posts from this blog

                    Can a sorcerer learn a 5th-level spell early by creating spell slots using the Font of Magic feature?

                    Does disintegrating a polymorphed enemy still kill it after the 2018 errata?

                    A Topological Invariant for $pi_3(U(n))$